Hermitian conjugate of the derivative of a wave function

In summary: Laroxe,The motivation for this definition comes from the idea of taking the adjoint of a linear transformation between vector spaces. Given two vector spaces V and W and a linear transformation T: V → W, the adjoint of T is a linear transformation T*: W → V such that for all vectors v in V and w in W,⟨v|T*w⟩ = ⟨Tv|w⟩.In quantum mechanics, we can view the Schrodinger equation as a linear transformation from the space of wavefunctions to the space of observables. The adjoint of this transformation is what we call the Hermitian conjugate, and it is defined using the same property as the general definition of the adjoint.The reason
  • #1
Gene Naden
321
64
I am continuing to work through Lessons on Particle Physics. The link is
https://arxiv.org/PS_cache/arxiv/pdf/0906/0906.1271v2.pdf
I am on page 22, equation (1.5.58). The authors are deriving the Hermitian conjugate of the Dirac equation (in order to construct the current). I am able to reproduce (1.5.58) except for one difference:
I have ##-i\gamma^0 \frac{\partial}{\partial t} \psi^\dagger - i\frac{\partial}{\partial x_k} \psi^\dagger (-\gamma^k)=0##
while the authors have ##-i\gamma^0 \frac{\partial}{\partial t} \psi^\dagger - i\frac{\partial}{\partial x^k} \psi^\dagger (-\gamma^k)=0##

I think, but I am not sure, that they are saying ##(\frac{\partial \psi}{\partial x_\mu})^\dagger=\frac{\partial \psi^\dagger}{\partial x^\mu}##

I would like clarification on whether this is in fact the root of my error. Thanks.
 
Physics news on Phys.org
  • #2
Am I going blind? I don't see any difference between the 2 equations.

(Btw, I would have thought that the ##\gamma^0## should be on the right of ##\psi^\dagger## in the 1st term.)
 
  • Like
Likes Gene Naden and vanhees71
  • #3
Gene Naden said:
I am continuing to work through Lessons on Particle Physics. The link is
https://arxiv.org/PS_cache/arxiv/pdf/0906/0906.1271v2.pdf
I am on page 22, equation (1.5.58). The authors are deriving the Hermitian conjugate of the Dirac equation (in order to construct the current). I am able to reproduce (1.5.58) except for one difference:
I have ##-i\gamma^0 \frac{\partial}{\partial t} \psi^\dagger - i\frac{\partial}{\partial x_k} \psi^\dagger (-\gamma^k)=0##
while the authors have ##-i\gamma^0 \frac{\partial}{\partial t} \psi^\dagger - i\frac{\partial}{\partial x^k} \psi^\dagger (-\gamma^k)=0##

I think, but I am not sure, that they are saying ##(\frac{\partial \psi}{\partial x_\mu})^\dagger=\frac{\partial \psi^\dagger}{\partial x^\mu}##

I would like clarification on whether this is in fact the root of my error. Thanks.

Since [itex]x^\mu[/itex] is real,

[itex](\frac{\partial}{\partial x^\mu} \psi)^\dagger = \frac{\partial}{\partial x^\mu} \psi^\dagger[/itex]

What they're saying is a fact about the gamma matrices (at least in their usual representation):

[itex](\gamma^\mu)^\dagger = \gamma^0 \gamma^\mu \gamma^0[/itex]

This can be proved by the four facts:
  1. [itex](\gamma^0)^2 =1[/itex]
  2. [itex]\gamma^0 \gamma^j = - \gamma^j \gamma^0[/itex] (with [itex]j=1,2,3[/itex])
  3. [itex](\gamma^0)^\dagger= \gamma^0[/itex]
  4. [itex](\gamma^j)^\dagger = -\gamma^j[/itex] (with [itex]j = 1,2,3[/itex])
So start with the Dirac equation:

[itex]i \gamma^\mu \frac{\partial}{\partial x^\mu} \psi = m \psi[/itex]

Take the conjugate:

[itex]-i \frac{\partial}{\partial x^\mu} \psi^\dagger (\gamma^\mu)^\dagger = m \psi^\dagger[/itex]

Rewrite [itex](\gamma^\mu)^\dagger[/itex] as [itex]\gamma^0 \gamma^\mu \gamma^0[/itex]:

[itex]-i \frac{\partial}{\partial x^\mu} \psi^\dagger \gamma^0 \gamma^\mu \gamma^0 = m \psi^\dagger[/itex]

Multiply on the right by [itex]\gamma^0[/itex]:

[itex]-i \frac{\partial}{\partial x^\mu} \psi^\dagger \gamma^0 \gamma^\mu \gamma^0 \gamma^0 = m \psi^\dagger \gamma^0[/itex]

Now, use that [itex](\gamma^0)^2 = 1[/itex]

[itex]-i \frac{\partial}{\partial x^\mu} \psi^\dagger \gamma^0 \gamma^\mu = m \psi^\dagger \gamma^0[/itex]

Finally, define [itex]\psi^\dagger \gamma^0 \equiv \bar{\psi}[/itex]

[itex]-i \frac{\partial}{\partial x^\mu} \bar{\psi} \gamma^\mu = m \bar{\psi}[/itex]

(That paper has a mistake in equation 1.5.58. The first term on the left should be [itex]-i \partial_t \psi^\dagger \gamma^0[/itex], not [itex]-i \gamma^0 \partial_t \psi^\dagger[/itex].)
 
  • Like
Likes Gene Naden
  • #4
Hi Stevendaryl,
Thanks for your input.

I am still wondering; what are the relations between ##\partial_{x_\mu}##, (##\partial_{x_\mu})^\dagger## and ##\partial^{x_\mu}##?

By multiplying it out, I have satisfied myself that ##-i \partial_t \psi^\dagger \gamma^0 = -i \gamma^0 \partial_t \psi^\dagger##. It is because ##\gamma^0## is symmetric and ##\partial_t \psi^\dagger## is a vector rather than a matrix.
 
  • #5
strangerep,
Thanks for taking an interest in my thread.
There is a difference; I had
##\frac{\partial}{\partial x_k}##
while they had
##\frac{\partial}{\partial x^k}##
I am wondering, however, if this difference matters. But since ##x^\mu=g^{\mu\sigma}x_{\sigma}##, it seems to make a difference.

See my earlier post where I argue that ##-i \partial_t \psi^\dagger \gamma^0 = -i \gamma^0 \partial_t \psi^\dagger##
 
  • #6
Gene Naden said:
Hi Stevendaryl,
Thanks for your input.

I am still wondering; what are the relations between ##\partial_{x_\mu}##, (##\partial_{x_\mu})^\dagger## and ##\partial^{x_\mu}##?

By multiplying it out, I have satisfied myself that ##-i \partial_t \psi^\dagger \gamma^0 = -i \gamma^0 \partial_t \psi^\dagger##. It is because ##\gamma^0## is symmetric and ##\partial_t \psi^\dagger## is a vector rather than a matrix.

In the usual way of doing the Dirac equation, [itex]\psi[/itex] is a column matrix, and [itex]\psi^\dagger[/itex] is a row matrix, and [itex]\gamma^0[/itex] is a square matrix. So you can't multiply [itex]\gamma^0 \psi^\dagger[/itex]. If [itex]A[/itex] is a matrix with [itex]n[/itex] columns and [itex]m[/itex] rows, and [itex]B[/itex] is a matrix with [itex]a[/itex] columns and [itex]b[/itex] rows, then [itex]A B[/itex] only makes sense if [itex]n = b[/itex].
 
  • Like
Likes Gene Naden
  • #7
Yes, Stevendaryl, I see what you mean. Now I need to review my derivation of 1.5.58. I must have gotten confused. Thanks for asserting reality for me!
 
  • #8
There is something a little confusing about the [itex]\dagger[/itex] operator when applied to the Dirac equation. In the case of the Schrodinger equation, [itex]\dagger[/itex] means Hermitian conjugate, which has the definition:

[itex]\langle \psi | A^\dagger \phi\rangle = \langle A \psi|\phi \rangle[/itex]

So according to this definition, [itex](\partial_x)^\dagger = - \partial_x[/itex], because

[itex]\langle (-\partial_x \psi) | \phi \rangle= - \int dx \frac{d \psi^*}{dx} \phi = - \int dx [\frac{d}{dx} (\psi^* \phi) - \psi^* \frac{d\phi}{dx}] = + \int dx \psi^* \frac{d\phi}{dx}= \langle \psi| \partial_x \phi\rangle[/itex] (the term [itex]\int dx \frac{d}{dx} (\psi^* \phi)[/itex] is zero).

But in the case of the Dirac equation, [itex]\dagger[/itex] means the complex conjugate of the transpose of a matrix. Since [itex]\partial_x[/itex] is neither a matrix, nor imaginary, [itex](\partial_x)^\dagger = \partial_x[/itex].

Perhaps someone with more mathematical knowledge than me can give more details about the relationship between [itex]\dagger[/itex] for matrices and [itex]\dagger[/itex] for Hilbert space elements?
 
  • #9
Stevendaryl, I see what you mean that ##-\langle \psi | (\partial_x) ^ \dagger \phi \rangle = \langle (-\partial_x \psi) | \phi \rangle= \langle \psi| \partial_x \phi\rangle##based on the definition
stevendaryl said:
⟨ψ|A†ϕ⟩=⟨Aψ|ϕ⟩

Can you please give me some motivation, or a reference, for this definition?

Thanks
 
Last edited:
  • #10
Gene Naden said:
Stevendaryl, I see what you mean that ##-\langle \psi | (\partial_x) ^ \dagger \phi \rangle = \langle (-\partial_x \psi) | \phi \rangle= \langle \psi| \partial_x \phi\rangle##based on the definitionCan you please give me some motivation, or a reference, for this definition?

Thanks

It's described here: https://quantummechanics.ucsd.edu/ph130a/130_notes/node133.html
 
  • #11
Thank you
 
  • #12
Gene Naden said:
There is a difference; I had
##\frac{\partial}{\partial x_k}##
while they had
##\frac{\partial}{\partial x^k}##
Oh crap -- I am going blind.

I am wondering, however, if this difference matters. But since ##x^\mu=g^{\mu\sigma}x_{\sigma}##, it seems to make a difference.
Yes, it does matter. The ##k## in ##\frac{\partial}{\partial x^k}## counts as a "downstairs" index. Hence it can be validly contracted with an upstairs index, as in ##\gamma^k##.

Otoh, in ##\frac{\partial}{\partial x_k}## the ##k## counts as an upstairs index. You can't validly contract 2 upstairs or 2 downstairs indices.

See my earlier post where I argue that ##-i \partial_t \psi^\dagger \gamma^0 = -i \gamma^0 \partial_t \psi^\dagger##
##\gamma^0## is a 4x4 matrix. ##\psi## is a 4-component column spinor. ##\psi^\dagger## is a 4-component row spinor.

Think about ordinary matrices and vectors. If you take a column vector ##v##, what does the transpose ##v^T## look like? Similarly, if you have an ordinary square matrix ##M##, what does ##(Mv)^T## look like? :oldwink:
 
  • #13
Thanks for your response, strangerep.

Yeah, ##\gamma^0 \partial_t \psi^\dagger## is just nonsense, as stevendaryl pointed out. Sorry about that.
 

1. What is the Hermitian conjugate of the derivative of a wave function?

The Hermitian conjugate of the derivative of a wave function is also known as the adjoint operator. It is the complex conjugate of the derivative of a wave function, where the sign of the imaginary part is reversed. It is represented by the symbol "†".

2. Why is the Hermitian conjugate important in quantum mechanics?

The Hermitian conjugate is important in quantum mechanics because it allows for the calculation of crucial physical quantities, such as expectation values and probabilities. It is also used in the construction of Hermitian operators, which are essential for describing physical observables in quantum mechanics.

3. How is the Hermitian conjugate of the derivative of a wave function related to the Schrödinger equation?

The Hermitian conjugate of the derivative of a wave function is directly related to the Schrödinger equation through the concept of the adjoint operator. The Schrödinger equation describes the time evolution of a quantum system, and the adjoint operator plays a crucial role in this process.

4. Can the Hermitian conjugate of the derivative of a wave function be represented mathematically?

Yes, the Hermitian conjugate of the derivative of a wave function can be represented mathematically using the following formula: (dψ/dx)† = (dψ*/dx) - (dψ/dx)*. This represents the complex conjugate of the derivative of a wave function, where the sign of the imaginary part is reversed.

5. What is the physical interpretation of the Hermitian conjugate of the derivative of a wave function?

The physical interpretation of the Hermitian conjugate of the derivative of a wave function is that it allows for the calculation of physical quantities such as expectation values and probabilities. In quantum mechanics, these quantities are related to the observable properties of a system, and the Hermitian conjugate plays a crucial role in their calculation.

Similar threads

  • Quantum Physics
Replies
9
Views
889
Replies
1
Views
568
  • Quantum Physics
Replies
21
Views
1K
Replies
29
Views
4K
  • Quantum Physics
Replies
1
Views
616
Replies
17
Views
1K
Replies
3
Views
831
  • Quantum Physics
Replies
13
Views
1K
Replies
24
Views
538
Replies
4
Views
1K
Back
Top